Question

In the figure below, assume that the current 1, is larger than the current 12 In parts a and b, decide whether there are places at which the total magnetic field is zero. State whether these places are located to the left of both wires, between the wires, or to the right of both wires. Wire 1 Wire 2 LF 2 Repulsion Attraction O O O (a) between the wires (b) between the wires (a) to the right of both wires (b) between the wires (a) to the left of both wires (b) between the wires

0 0
Add a comment Improve this question Transcribed image text
Answer #1

For figure a.

Field due to I1 to the right of the wire will be outward the page whereas due to I2 right to the wire is inward.

Hence zero net field may exists right of the both wire.

Option second is correct.

Add a comment
Know the answer?
Add Answer to:
In the figure below, assume that the current 1, is larger than the current 12 In...
Your Answer:

Post as a guest

Your Name:

What's your source?

Earn Coins

Coins can be redeemed for fabulous gifts.

Not the answer you're looking for? Ask your own homework help question. Our experts will answer your question WITHIN MINUTES for Free.
Similar Homework Help Questions
  • Two parallel wires carry electric current in the same direction as shown in the figure to...

    Two parallel wires carry electric current in the same direction as shown in the figure to the right. The current in each wire produces a magnetic field, and each wire experiences a magnetic force due to the field produced by the other wire In the region of space near wire 1, in what direction does the magnetic field produced by wire 2 point? O up O down O right O left into the screen O out of the screen In...

  • The figure below shows two wires carrying currents of I = 4.75 A in opposite directions,...

    The figure below shows two wires carrying currents of I = 4.75 A in opposite directions, separated by a distance d. Assume d = 11.0 cm. P2 2d (a) Find the magnitude (in pT) and direction of the net magnetic field at a point halfway between the wires. magnitude T direction into the page (b) Find the magnitude (in yT) and direction of the net magnetic field at point P1, 11.0 cm to the right of the wire on the...

  • PLEASE HELP!!! 8. Two long, parallel wires each carry a current I in the same direction...

    PLEASE HELP!!! 8. Two long, parallel wires each carry a current I in the same direction as shown in the figure. The magnetic field at the point P, midway between the two wires is ________. zero directed into the page directed out of the page parallel to the wires in the same direction as the current. parallel to the wires, in the opposite direction as the current. 9. A long, straight wire carries a current of 20A to the right....

  • Consider two current-carrying wires, separated by a distance d =3.1 cm, as shown in the figure.

    Consider two current-carrying wires, separated by a distance d =3.1 cm, as shown in the figure. The left wire is directed out of the page with current Iy, and the right wire is directed into the page with current I2. The point P is a distance d from both wires, so the wires and the point form an equilateral triangle. Part (a) If both wires are carrying a current of 9.5 A. what is the magnitude of the magnetic field, in...

  • As shown in the figure, two long, straight current-carrying wires (1 and 2) are attached at right angles to a board by wire brackets. The current in wire 1 is 16.1 A in the negative y directio...

    As shown in the figure, two long, straight current-carrying wires (1 and 2) are attached at right angles to a board by wire brackets. The current in wire 1 is 16.1 A in the negative y direction and the current in wire 2 is 5.00 A in the positive x direction. The points A and B are a distance r 0.200 m from both wires. #1 #2 (a) How does tia magnitude of the net magnetic field at point A...

  • The figure below shows a cross-section of two long, straight, parallel wires, where the currents are flowing perpendicul...

    The figure below shows a cross-section of two long, straight, parallel wires, where the currents are flowing perpendicular to the page. Wire 1 carries a current I1 directed into the page (in the −z direction) and passes through the x-axis at x = +a. Wire 2 passes through the x-axis at x = −2a and carries an unknown current I2. The total magnetic field at the origin due to the current-carrying wires has the magnitude 2μ0I1 (2πa) . Note that...

  • electrostatic charging quiz As shown in the figure below, two lono parallel wires (1 and 2)...

    electrostatic charging quiz As shown in the figure below, two lono parallel wires (1 and 2) carry currents of I, -2.90 A and 12 - 4.95 A in the direction indicated. (a) Determine the magnitude and direction of the magnetic field at a point midway between the wires (d magnitude 82PT direction 90 10.0 cm). o counterclockwise from the tx-axis (b) Determine the magnitude and direction of the magnetic field at point P, located d 10.0 cm above wire 1...

  • 1. The figure on the left shows the current, 1, of a long straight wire coming...

    1. The figure on the left shows the current, 1, of a long straight wire coming out of the page. The figure on the right shows the direction of the magnetic field as it passes through the plane of the page for a vertical current-carrying wire (center) For the figure on the left draw the magnetic field of the wire; for the figure on the right indicate the direction of the current. O O Bout I (out)

  • As shown in the figure, two long, straight current-carrying wires (1 and 2) are attached right...

    As shown in the figure, two long, straight current-carrying wires (1 and 2) are attached right angles to a board by wire brackets. The current in wire 1 is 6.73A in the negative y direction and the current in wire 2 is 5.00 A in the positive x direction. The points and are a distance - 0.200 m from both wires. (*) How does the magnitude of the net magnetic field at point A compare to the magnitude of the...

  • Hi! Please help solve for questions 5 and 6! Thank you so much The figure below...

    Hi! Please help solve for questions 5 and 6! Thank you so much The figure below shows two infinitely long, straight, current-carrying wires located at x = -20.0 cm and at x = +30.0 cm. The left-hand wire carries current z direction (into the page), and the current I_2 in the right-hand wire is unknown. If the magnitude of the total magnetic field due to the two wires at the origin is 3.60 mu T, what are the two possible...

ADVERTISEMENT
Free Homework Help App
Download From Google Play
Scan Your Homework
to Get Instant Free Answers
Need Online Homework Help?
Ask a Question
Get Answers For Free
Most questions answered within 3 hours.
ADVERTISEMENT
ADVERTISEMENT
ADVERTISEMENT